Help with Deriving d/dt |r(t)| = (1/|r(t)|) r(t)dot r'(t)

  • Thread starter Thread starter bubbers
  • Start date Start date
Click For Summary
SUMMARY

The discussion focuses on deriving the equation d/dt |r(t)| = (1/|r(t)|) (r'(t) dot r(t)) from the vector function r(t). The key hint provided is that |r(t)|^2 = r(t) dot r(t). The correct application of the chain rule leads to the conclusion that the derivative of the magnitude of r(t) is dependent on both the vector and its derivative. The final expression confirms the relationship between the derivative of the magnitude and the dot product of the vector and its derivative.

PREREQUISITES
  • Understanding of vector calculus
  • Familiarity with the chain rule in differentiation
  • Knowledge of dot products in vector analysis
  • Basic concepts of vector functions and their magnitudes
NEXT STEPS
  • Study the chain rule in depth, particularly in the context of vector functions
  • Learn about the properties of dot products and their applications in physics
  • Explore examples of differentiating vector magnitudes in various contexts
  • Investigate the implications of vector derivatives in motion analysis
USEFUL FOR

Students studying calculus, particularly those focusing on vector calculus, as well as educators and tutors looking to clarify concepts related to vector derivatives and magnitudes.

bubbers
Messages
6
Reaction score
0

Homework Statement



If vector r(t) is not 0, show that d/dt |r(t)| = (1/|r(t)|) r(t)dot r'(t).

Homework Equations



The hint given was that |r(t)|^2 = r(t) dot r(t)

The Attempt at a Solution



the post I saw about this question said that you should take the derivatives of both sides of the hint equation using chain rule, so I got:

2|r(t)|(1)=r'(t) dot r(t) + r(t) dot r'(t)
which is equal to:
|r(t)|= r'(t) dot r(t)

aaaaand now I don't know what to do
 
Last edited:
Physics news on Phys.org


bubbers said:
2|r(t)|(1)=r'(t) dot r(t) + r(t) dot r'(t)

This step is incorrect. You have the equivalent of f(g(x)) -- the chain rule says d/dx f(g(x)) = f'(g(x)) * g'(x).
 


Okay, now I'm confused... in this situation, wouldn't f'(g(x)) be equivalent to 2|r(t)| and g'(x) be equivalent to 0 because |r(t)| is a length and the derivative of a number is 0? So then it would be 2|r(t)|(0)=r'(t) dot r(t) + r(t) dot r'(t) --> 0=r'(t) dot r(t) + r(t) dot r'(t)?
If this is right, then I still don't know where to go from here.
 


bubbers said:
Okay, now I'm confused... in this situation, wouldn't f'(g(x)) be equivalent to 2|r(t)| and g'(x) be equivalent to 0 because |r(t)| is a length and the derivative of a number is 0? So then it would be 2|r(t)|(0)=r'(t) dot r(t) + r(t) dot r'(t) --> 0=r'(t) dot r(t) + r(t) dot r'(t)?
If this is right, then I still don't know where to go from here.

No. |r(t)| has t dependence. Accordingly, we can't assume the magnitude is constant.
 


Oh man...I totally get it now...i feel kinda dumb :P
the derivative would just be:
2|r(t)|d/dt|r(t)|= r'(t) dot r(t) + r(t) dot r'(t)
2|r(t)|d/dt|r(t)|= 2(r'(t) dot r(t))
|r(t)|d/dt|r(t)|= r'(t) dot r(t)
d/dt|r(t)|=(1/|r(t)|) ( r'(t) dot r(t))

Thanks for letting me waste your time :)
 


No problem. Glad to have helped. :)
 
Question: A clock's minute hand has length 4 and its hour hand has length 3. What is the distance between the tips at the moment when it is increasing most rapidly?(Putnam Exam Question) Answer: Making assumption that both the hands moves at constant angular velocities, the answer is ## \sqrt{7} .## But don't you think this assumption is somewhat doubtful and wrong?

Similar threads

Replies
6
Views
1K
  • · Replies 3 ·
Replies
3
Views
2K
Replies
5
Views
2K
Replies
0
Views
1K
  • · Replies 1 ·
Replies
1
Views
2K
Replies
1
Views
1K
  • · Replies 1 ·
Replies
1
Views
1K
  • · Replies 3 ·
Replies
3
Views
2K
  • · Replies 5 ·
Replies
5
Views
2K
  • · Replies 5 ·
Replies
5
Views
3K